8
$\begingroup$

Apologies if this has been asked before. I am wondering if the following series has a closed form:

$$\alpha=\sum_{n=1}^\infty\frac{W(n^2)}{n^2}\tag{1}$$

where $W(x)$ is the Lambert W function. I am interested in this series as an extension of the series:

$$\sum_{n=1}^\infty\frac{\ln{n^2}}{n^2}=-2\zeta'(2)=\frac{\pi^2}{3}\ln{\left(\frac{A^{12}}{2\pi e^\gamma}\right)}\tag{2}$$

since $W(x)$ is the product logarithm. Obviously $(1)$ must converge by comparison with $(2)$ since $W(x)e^{W(x)}=x$ so $W(x)e^{W(x)}<\ln(x)e^{\ln{x}}$ so $W(x)<\ln{x}$ for $x\ge1$ by monotonicity of $W(x)$. However, I am not sure what it converges to. A value (obtained by a couple of toy formal methods) is $\alpha\overset{!}{=}\sqrt{2\pi}-\frac{1}{2},ドル but I do not think this is an equality (although it is hard to tell since the series converges so slowly). Using Wolfram Alpha to sum to 10000ドル$ terms, the partial sum appears to be 2ドル.0142453>2.0066283=\sqrt{2\pi}-\frac{1}{2},ドル but I do not know $\alpha$'s exact numerical value.

Thus my question is: Is there a closed form for $\alpha$? If not, is there an expression for the error in the $\sqrt{2\pi}-\frac{1}{2}$ approximation?


My attempts: (Note: I have only put the following here to show where I got the value of $\sqrt{2\pi}-\frac{1}{2}$ from; I assume that the methods are not properly correct). In the first place, using this toy resummation formula I had derived, the formula $\int_0^\infty\frac{W(x^2)}{x^2}dx=\sqrt{2\pi}$ (derivable from $\int_0^\infty\frac{W(x)}{x\sqrt{x}}dx=\sqrt{8\pi}$) and the Taylor series of $W(x)$ I formally calculated $\alpha=\sqrt{2\pi}-\frac{1}{2}$. However, the given resummation formula often gives the wrong answer.

I also attempted to evaluate the integral using the Abel-Plana formula (although I do not think $\frac{W(z^2)}{z^2}$ satisfies the conditions to apply it). Formally using the fact that $\lim\limits_{s\rightarrow0}\frac{W(s^2)}{s^2}=1,ドル I got:

$$\sum_{n=1}^\infty\frac{W(n^2)}{n^2}=\int_{0}^\infty\frac{W(x^2)}{x^2}\;dx-\frac{1}{2}+i\int_0^\infty\frac{\frac{W(-t^2)}{-t^2}-\frac{W(-t^2)}{-t^2}}{e^{2\pi t}-1}\;dt=\sqrt{2\pi}-\frac{1}{2}$$

I assume that the occurrences of this incorrect value are to do with my incorrect applications of these rules missing some remainder term, but I do not know a good way of rectifying this. So my question is: does anyone know a way of evaluating $(*)$ exactly?

asked Feb 8, 2017 at 2:31
$\endgroup$
20
  • 2
    $\begingroup$ This is a really strange question, but a closed form might exist. For instance, $\frac{W(n)}{n^2}$ is the residue at $z=W(n)$ of $\frac{\pi(1+z)\cot(\pi z e^z)}{z e^z}$. However, I do not think the hypothesis of EMC or Abel-Plana are met, and numerically $\sum_{n\geq 1}\frac{W(n^2)}{n^2}$ is a bit larger than $\sqrt{2\pi}-\frac{1}{2},ドル even if not by much. $\endgroup$ Commented Feb 8, 2017 at 3:04
  • 6
    $\begingroup$ The issue is that $W(z^2)/z^2$ has branch cut along the imaginary axis. I guess that resolving this branch-cut issue yields $$\sum_{n=1}^{\infty}\frac{W(n^2)}{n^2} = \sqrt{2\pi} - \frac{1}{2} + 2 \int_{1/\sqrt{e}}^{\infty} \frac{\operatorname{Im}W(-t^2+i 0^+)}{(e^{2\pi t} - 1) t^2} ,円 dt. $$ Mathematica 11 seems to be agreeing with this claim that both sides give approx. values 2ドル.016001\cdots$. $\endgroup$ Commented Feb 8, 2017 at 3:09
  • 1
    $\begingroup$ Running millions of summations through mathematica gives an answer of about 2ドル.016$ $\endgroup$ Commented Feb 8, 2017 at 3:09
  • 1
    $\begingroup$ It just seems to me a pure evil. :s $\endgroup$ Commented Feb 8, 2017 at 3:12
  • 1
    $\begingroup$ @JackD'Aurizio, I take my word back. Honestly I have no idea, but now I am more convinced that it is bounded by $\pi$. $\endgroup$ Commented Feb 8, 2017 at 3:34

1 Answer 1

4
$\begingroup$

Just a few numbers

Computing $$S_k=\sum_{n=1}^{10^k}\frac{W\left(n^2\right)}{n^2}$$ $$\left( \begin{array}{cc} k & S_k \\ 3 & 2.00276605784206527342788 \\ 4 & 2.01424531490397345666972 \\ 5 & 2.01578185484283675359672 \\ 6 & 2.01597490197032815619980 \\ 7 & 2.01599818252359384501875 \\ \end{array} \right)$$

The last value seems to be quite close to $$\frac{1}{\gamma }\sqrt{\Gamma \left(\frac{2}{3}\right)}=S_7+3.775\times 10^{-9}$$

answered Sep 1, 2023 at 3:09
$\endgroup$

You must log in to answer this question.

Start asking to get answers

Find the answer to your question by asking.

Ask question

Explore related questions

See similar questions with these tags.